Saltar al contenido

Encuentre todos los pares de enteros positivos $ (a, b) $ tales que $ 2 ^ a + 5 ^ b $ sea un cuadrado perfecto.

Buscamos por el mundo online para así traerte la respuesta para tu duda, si continúas con alguna difcultad deja tu duda y te contestaremos sin falta, porque estamos para servirte.

Si $ 2 ^ a + 5 ^ b = w ^ 2 $, entonces $ 2 ^ a equiv w ^ 2 pmod 5 $. Esto implica que $ 2 mid a $. Entonces deja $ c in Bbb N espacio | espacio 2c = a $.
$$ 2 ^ 2c + 5 ^ b = w ^ 2 $$$$ 5 ^ b = w ^ 2-2 ^ 2c $$$$ 5 ^ b = (w-2 ^ c) (w + 2 ^ c) $$
ambos $ w-2 ^ c $ y $ w + 2 ^ c $ son ambos poderes de $ 5 $ entonces $ existe espacio s, t en Bbb N espacio | espacio b = s + t, s> t, espacio 5 ^ s = w + 2 ^ c, 5 ^ t = w-2 ^ c $$$ 5 ^ s = w + 2 ^ c espacio espacio espacio (1) $$$$ 5 ^ t = w-2 ^ c espacio espacio espacio (2) $$
restando $ (2) $ desde $ (1) $ obtenemos
$$ 5 ^ s-5 ^ t = 2 ^ c + 1 espacio espacio espacio (3) $$$$ 5 ^ t (5 ^ st -1) = 2 ^ c + 1 espacio espacio espacio (4) $$$ 5 nmid 2 ^ c + 1 $ entonces $ t = 0 $ y $ s = b $. Por lo tanto
$$ 5 ^ b-1 = 2 ^ c + 1 espacio espacio espacio (5) $$$$ 5 ^ b-5 + 2 ^ 2 = 2 ^ c + 1 espacio espacio espacio (6) $$$$ 5 ^ b-5 = 2 ^ c + 1 -2 ^ 2 espacio espacio espacio (7) $$$$ 5 (5 ^ b-1 -1) = 2 ^ 2 (2 ^ c-1 -1) espacio espacio espacio (8) $$$ 5 mid $lhs de (8) $ Flecha derecha $$ 5 mid $dcha. de (8). $ 5 nmid 2 ^ 2 $ entonces $ 5 mid 2 ^ c-1 -1 Flecha derecha 4 mid c-1 $

lhs de (8) $ equiv 0,20,27 pmod 31 $ y dcha. de (8) $ equiv 0,4,12,28,29 pmod 31 $

La única forma de que la ecuación sea igual es si ambos lados son divisibles entre 31.

$ 31 nmid 2 ^ 2 $ por lo tanto $ 31 mid 2 ^ c-1 -1 Flecha derecha 5 mid c-1 $

Combinando ambos hechos que $ 4 mid c-1 $ y $ 5 mid c-1 Rightarrow 20 mid c-1 Rightarrow 25 mid 2 ^ c-1 -1 Rightarrow 25 mid $dcha. de (8)$ Flecha derecha 25 mid $lhs de (8) $ Flecha derecha 5 mid 5 ^ b-1 -1 Flecha derecha b = 1 $

enchufar $ 1 $ por $ b $ en $ (5) $ implica que $ c = 1 Flecha derecha a = 2 $

la primera solución es $ a = 2 $, $ b = 1 $, $ w ^ 2 = 9 $

Editar: Al comienzo de la prueba asumí que $ 2 ^ a equiv w ^ 2 pmod 5 $. Esto es true si $ b neq 0 $.

Si $ b = 0 $ entonces $ 2 ^ a + 1 = w ^ 2 $$$ 2 ^ a + 1 = w ^ 2 $$$$ 2 ^ a = w ^ 2-1 $$$$ 2 ^ a = (w-1) (w + 1) $$
ambos $ w-1 $ y $ w + 1 $ son ambos poderes de $ 2 $ entonces $ existe espacio u, v en Bbb N espacio | espacio a = u + v, u> v, espacio 2 ^ u = w + 1,2 ^ v = w-1 $$$ 2 ^ u = w + 1 quad (9) $$$$ 2 ^ v = w-1 quad (10) $$
restando $ (10) $ desde $ (9) $ obtenemos
$$ 2 ^ u-2 ^ v = 2 quad (11) $$$$ 2 ^ v (2 ^ uv -1) = 2 quad (12) $$
Esto implica $ v = 1 $ y ambos lados de (12) se pueden dividir por 2
$$ 2 ^ u-1 -1 = 1 quad (13) $$$$ 2 ^ u-1 = 2 quad (14) $$$$ u-1 = 1 quad (15) $$$$ u = 2 quad (16) $$
Esto da la segunda solución. $ a = 3 $, $ b = 0 $, $ w ^ 2 = 9 $

Edición 2: demasiado tiempo para comentar, respondiendo a la pregunta de Prathmesh del comentario a continuación

Después de llegar a la ecuación $ (8) $ Sabía que si pudiera demostrar eso $ 5 mid 5 ^ b-1 -1 $ entonces solo habría una solución para esa parte del problema. Trabajando al revés $ 25 mid 2 ^ c-1 -1 $. poderes de $ 2 $ tener un ciclo de $ 20 pmod 25 $. Entonces $ 25 mid 2 ^ c-1 -1 $ si $ 20 mid c-1 $. Yo ya tenía $ 4 mid c-1 $ por el hecho de que ambos lados de $ (8) $ son divisibles por $ 5 $. Entonces necesitaba un mod que produzca un ciclo que sea un múltiplo de $ 5 $ en poderes de $ 2 $. En general, si un mod producirá un ciclo $ x $ si el mod es divisible por una potencia suficientemente grande de $ x $ o es divisible por un factor primo de la forma $ xk + 1 $. Entonces, el mod en este caso debe tener una potencia suficientemente grande de $ 5 $ o tiene un factor primo de la forma $ 5k + 1 $. Dado que estoy tratando de demostrar que el rhs es divisible por $ 25 $ No puedo usar poderes de $ 5 $ así que me quedo con números que tienen un factor primo de la forma $ 5k + 1 $. Todos los primos excepto $ 2 $ son impares, así que puedo mirar los números de la forma $ 10k + 1 $. Así que lo intenté primero $ 11 $ pero ambos lados pueden ser $ equiv 1,3,4,5,9 pmod 11 $. $ 21 $ tiene una factorización prima de $ 3 $ y $ 7 $ ninguno de los cuales es de la forma $ 10k + 1 $. entonces lo intenté $ 31 $.

Si $ 2 ^ a + 5 ^ b = m ^ 2 $, entonces $ m ^ 2 equiv 0, 1, 4 pmod 5 $. $ 0 $ es imposible ya que $ 2 ^ a $ nunca es divisible entre 5, por lo que $ 2 ^ a + 5 ^ b equiv 2 ^ a equiv 1, 4 pmod 5 $. Esto implica que $ a $ es incluso, digamos $ a = 2k $. Esto da $ 5 ^ b = m ^ 2-2 ^ 2k = (m-2 ^ k) (m + 2 ^ k) $, lo que implica que tanto $ m-2 ^ k $ y $ m + 2 ^ k $ son poderes de $ 5 $, decir
$$ m-2 ^ k = 5 ^ r, m + 2 ^ k = 5 ^ s, r
Pero $ 5 ^ s-5 ^ r $ es divisible por 5 para $ r> 0 $, por lo que no puede ser un poder de dos a menos que $ r = 0 $.

Por tanto, cualquier solución debe tener $ m = 2 ^ k + 1 $, $ m + 2 ^ k = 5 ^ b $, de modo que $ 5 ^ b = 2 ^ k + 1 + 1 $, o $ 5 ^ b-2 ^ k + 1 = 1 $. Según el teorema de Mihăilescu, ya sea $ b le 1 $ o $ k + 1 le 1 $. Ya que $ b $ es un número entero positivo, obtenemos $ b = 1 $, de donde $ 2 ^ k + 1 = 4 $ y $ k = 1 $, entonces $ a = 2 $.

valoraciones y reseñas

Agradecemos que desees proteger nuestro análisis exponiendo un comentario y dejando una puntuación te estamos agradecidos.

¡Haz clic para puntuar esta entrada!
(Votos: 0 Promedio: 0)


Tags :

Utiliza Nuestro Buscador

Deja una respuesta

Tu dirección de correo electrónico no será publicada. Los campos obligatorios están marcados con *